Jul 192019
 

2019 第 60 届 IMO 解答

Problem 1 ()

很稀松平常的方程. 令 \(a=0\), 于是

\[f(0) + 2f(b) = f(f(b)).\]

因此, 我们只要考察

\[f(2a) + 2f(b) = f(0) + 2f(a+b) \]

就成.

令 \(a=1\), 我们有

\[f(2) + 2f(b) = f(0) + 2f(1+b). \]

这也就是 \(f(b+1) – f(b) =\frac{f(2)-f(0)}2\). 从而, \(f(n)\) 是线性的. 设 \(f(n)=An+B\)(\(A\), \(B\) 是待定的常数). 结合 \(f(0) + 2f(n) = f(f(n))\) 可知

\[B+2(An+B)=A(An+B)+B.\]

于是 \(2A=A^2\), \(3B=AB+B\). 故而, \((A,B)=(0,0)\), \((2, k)\), 这里 \(k\) 是任意的整数.

经检验, \(f(n)=0\) 与 \(f(n)=2n+k\) 符合要求(\(k\) 是任意的整数常数).

综上所述, 所求的函数即是 \(f(n)=0\) 与 \(f(n)=2n+k\) (\(k\) 是任意的整数常数).

Problem 2 ()

 Posted by at 2:46 pm  Tagged with:
May 202018
 

2018 年IMO 国家队队员李一笑——来自江苏天一中学——的大作 “2018 年国家集训队第一阶段选拔试题及解答”. 文档转载自数学新星网.

2018 China IMO team selection test part one

2018 年国家集训队第二阶段选拔试题来自贴吧

2018 China IMO team selection test

2018 China IMO team selection test part two 1

2018 China IMO team selection test

2018 China IMO team selection test part two 2

2018 China IMO team selection test

2018 China IMO team selection test part two 3

2018 China IMO team selection test

2018 China IMO team selection test part two 4

 Posted by at 8:41 am
Nov 162017
 

不是完整的试题解答, 仅仅在关隘的地点聊一聊.

付云皓的题 3 的解

记 \(a=[nq^{\frac13}]\), \(b=[nq^{\frac23}]\), \(c=nq\). 然后

\begin{equation}\Big(c-aq^{\frac23}\Big)^2+\Big(c-bq^{\frac13}\Big)^2+\Big(aq^{\frac23}-bq^{\frac13}\Big)^2=\frac{2(a^3q^2+b^3q+c^3-3abcq)}{aq^{\frac23}+bq^{\frac13}+c}\geqslant\frac2{3c},\end{equation}

最后的不等式是因为 \(a^3q^2+b^3q+c^3\gt 3abcq\), 并且 \(c\geqslant aq^{\frac23}\), \(c\geqslant bq^{\frac13}\).

然后, 因为 \(c-aq^{\frac23}\geqslant0\), \(c- bq^{\frac13}\geqslant0\), 以及 \(aq^{\frac23}-bq^{\frac13}=-\Big((c-aq^{\frac23})-(c-bq^{\frac13})\Big)\), 得到

\begin{equation}\Big(c-aq^{\frac23}\Big)^2+\Big(c-bq^{\frac13}\Big)^2\leqslant\Big(2c-aq^{\frac23}-bq^{\frac13}\Big)^2,\end{equation}

\begin{equation}\Big(aq^{\frac23}-bq^{\frac13}\Big)^2\leqslant\Big(2c-aq^{\frac23}-bq^{\frac13}\Big)^2.\end{equation}

现在, \((1)\) 给出

\begin{equation}2\Big(2c-aq^{\frac23}-bq^{\frac13}\Big)^2\geqslant\frac2{3c},\end{equation}

记得 \(c=nq\), 这也就是

\begin{equation}\Big(q^{\frac13}\cdot \{nq^{\frac23}\}+q^{\frac23}\cdot \{nq^{\frac13}\}\Big)^2\geqslant\frac1{3nq}.\end{equation}

随即我们有

\begin{equation}\Big\{nq^{\frac23}\Big\}+ \Big\{nq^{\frac13}\Big\}\geqslant\frac1{q\sqrt{3qn}}.\end{equation}

然后是邓煜给出的第三题的答案, 从知乎转来.

 Posted by at 8:39 pm  Tagged with:
Nov 162017
 

第 33 届中国数学奥林匹克

浙江 杭州

第一天

(2017 年 11 月 15 日    8:00–12:30)

1. 设 \(A_n\) 是满足以下条件的素数 \(p\) 的集合: \(\exists a\), \(b\in\Bbb N^+\), 使得 \(\dfrac{a+b}p\), \(\dfrac{a^2+b^2}{p^2}\) 都是正整数, 且

\[\Big(\frac{a+b}p, p\Big)=\Big(\frac{a^2+b^2}{p^2}, p\Big)=1.\]

证明: (1) \(A_n\) 为有限集当且仅当 \(n\ne2\);

(2) 记 \(f(n)=|A_n|\). 若 \(k\), \(m\) 为正奇数, \(d=(k, m)\), 则

\[f(d)\leqslant  f(k)+f(m)-f(km)\leqslant 2f(d).\]

2. 设

\[T=\{(x, y, z)|1\leqslant x, y, z\leqslant n\}\]

为空间中 \(n^3\) 个点. 将其中 \((3n^2-3n+1)+k\) 个点染为红色, 且若 \(P\), \(Q\) 为红色, \(PQ\) 平行于任一条坐标轴, 则线段 \(PQ\) 上的所有整点均为红色. 求证: 至少有 \(k\) 个边长为 \(1\) 的立方体的所有顶点均为红色.

3. 设 \(n\), \(q\) 为正整数, \(q\) 不是完全立方数. 求证: 存在正实数 \(c\) 满足
\[\{nq^{\frac13}\}+\{nq^{\frac23}\}\geqslant \frac c{\sqrt n}\]

对所有正整数 \(n\) 成立, 其中 \(\{\cdot\}\) 表示其小数部分.

第 33 届中国数学奥林匹克

浙江 杭州

第二天

(2017 年 11 月 16 日    8:00–12:30)

4. 已知圆内接四边形 \(ABCD\), 其对角线 \(AC\) 与 \(BD\) 交于 \(P\) 点, \(\triangle ADP\) 的外接圆交 \(AB\) 于 \(E\), \(\triangle BCP\) 的外接圆交 \(AB\) 于 \(F\). \(\triangle ADE\) 与 \(\triangle BCF\) 的内心分别为 \(I\), \(J\), 直线 \(IJ\) 交 \(AC\) 于 \(K\).
求证: \(A\), \(I\), \(K\) , \(E\) 四点共圆.

cmo 2017

cmo 2017 p4

5. 对 \(n\times n\) 的方格进行黑白染色, 若两个方格 \(a\), \(b\) 有公共顶点且同色, 则称 \(a\), \(b\) 这两个方格相邻. 若 \(a\), \(b\) 能通过一系列的方格 \(c_1\to c_2\to\dotsb\to c_k\), 其中 \(c_1=a\), \(c_2=b\), 且 \(c_i\), \(c_{i+1}\) 相邻, 则称 \(a\), \(b\) 连通. 求最大正整数 \(M\), 使得存在 \(M\) 个方格, 使得其两两不连通.

6. 给定正整数 \(n\), \(k\), \(n\gt k\), 其中 \(a_i\in (k-1, k)\), \(1\leqslant i\leqslant n\). 若正实数 \(x_1\), \(x_2\), \(\dotsc\),\(x_n\) 满足: 对任意集合 \(I\subset\{1,2,\dotsc,n\}\), \(|I|=k\) 有\(\sum\limits_{i\in I}x_i\leqslant \sum\limits_{i\in I}a_i\), 试求 \(\prod\limits_{i=1}^nx_i\) 的最大值.

 Posted by at 8:05 pm  Tagged with:
Jul 182017
 

2017 第 58 届 IMO 解答

Problem 1 (South Africa)

Lemma 1 当实数 \(x\geqslant6\), 则 \(x\lt(x-3)^2\).

Lemma 2 若整数 \(m\) 符合 \(m \equiv 2\pmod3\), 则 \(m\) 不能是完全平方.

若数列 \(\{a_n\}\) 的某一项 \(\gt1\), 则紧挨着的后一项也 \(\gt1\). 因此, \(a_0\gt1\) 说明这数列的所有项都 \(\gt1\).

首先, 若数列 \(\{a_n\}\) 的某一项 \(a_i\) 使得 \(a_i\equiv 2\pmod3\), 则这一项不是完全平方, \(a_{i+1}=a_i+3\), \(a_{i+1}\equiv 2\pmod3\). 于是, \(a_0\equiv 2\pmod3\) 之时, 数列 \(\{a_n\}\) 就是一个以 \(a_0\) 为首项, 公差为 \(3\) 的等差数列. 此时, 数列 \(\{a_n\}\) 的各项互不相同导出如此这般的 \(a_0\) 不是我们寻找的. 进而, 数列 \(\{a_n\}\) 的如果有一项 \(\equiv 2\pmod3\), 那么这数列不可能有无穷多项相同.

其次: 在数列 \(\{a_n\}\) 中, 如果存在某一项 \(a_s(s\geqslant0)\) 使得 \(a_s \equiv 0, 1\pmod3\), 且 \(a_s\geqslant6\), 那么必定有大于 \(s\) 的正整数 \(t\), 使得 \(a_t\lt a_s\), 并且当 \(a_s \equiv 0\pmod3\) 时, \(a_t\equiv 0\pmod3\); 当 \(a_s \equiv 1\pmod3\) 时, \(a_t\not\equiv 0\pmod3\).

这是因为, 在 \(a_s \equiv 0, 1\pmod3\), \(a_s\geqslant6\) 时, 我们依次考察从 \(a_s\) 开始的项:

\begin{equation}a_s, a_{s+1}, a_{s+2}, \dotsc.\end{equation}

然后

\[a_s\lt(a_s-3)^2,\qquad a_s \equiv (a_s-3)^2\pmod3\]

表明 \((1)\) 中必有一项 \(a_u(u\geqslant s)\), 满足 \(a_u\) 为完全平方, 且 \(a_u\leqslant (a_s-3)^2\). 取 \(t=u+1\), 则 \(t\gt s\), \(a_t=a_{u+1}\leqslant a_s-3\) 蕴涵 \(t\) 符合我们的要求.

现在, 当 \(a_0\equiv 0, 1\pmod3\), 我们知道必有 \(\{a_n\}\) 的某一项 \(a_j(j\geqslant0)\), 使得 \(a_j\lt6\), 并且 \(a_0 \equiv 0\pmod3\) 时, \(a_j\equiv 0\pmod3\); 当 \(a_0 \equiv 1\pmod3\) 时, \(a_j\not\equiv 0\pmod3\).

事实上, 如果 \(a_0\geqslant6\), 那么必定有大于正整数  \(t_1\), 使得 \(a_{t_1}\lt a_0\); 在 \(a_{t_1}\geqslant6\), 那么必定有大于 \(t_1\) 的正整数 \(t_2\), 使得 \(a_{t_2}\lt a_{t_1}\); 在 \(a_{t_2}\geqslant6\), 有大于 \(t_2\) 的正整数 \(t_3\), 使得 \(a_{t_3}\lt a_{t_2}\);…; 如此这般下去. 但这个过程不可能一直继续: 我们最终会得到 \(\{a_n\}\) 的某一项 \(a_j(j\geqslant0)\), 使得 \(a_j\lt6\).

既然 \(a_j\), 使得 \(1\lt a_j\lt6\), 于是 \(a_j\) 只可能是 \(2\), \(3\), \(4\),  \(5\).

当 \(a_j=4\) 时, \(a_{j+1}=2\). 于是, 在 \(a_j\) 为 \(2\), \(4\), \(5\) 之一(此时 \(a_0 \equiv 1\pmod3\)), 当 \(k\gt j\), 有 \(a_k\equiv 2\pmod3\). 因之, 数列 \(a_{j+1}\), \(a_{j+2}\), \(a_{j+2}\), \(\dotsc\) 是一个以 \(a_{j+1}\) 为首项, 公差为 \(3\) 的等差数列, 故而不可能在数列 \(\{a_n\}\) 有无穷多项重复.

当 \(a_j=3\) 时(此时 \(a_0 \equiv 0\pmod3\)), \(a_{j+1}=6\), \(a_{j+2}=9\), \(a_{j+3}=3\). 于是, 数列 \(\{a_n\}\) 从 \(a_j\) 开始的项依次就是

\[3, 6, 9, 3, 6, 9, 3, 6, 9, \dotsc.\]

此时, \(3\), \(6\), \(9\) 都会在数列 \(\{a_n\}\) 无穷多次的重复出现,

综上所述, 我们寻找的符合要求的所有 \(a_0\) 就是全体的满足 \(a_0 \equiv 0\pmod3\) 的正整数.

Problem 2 (Dorlir Ahmeti, Albania)

本题的难点是 \(f(x)\) 为单射.

如果\(f(0)=0\).

令 \(y=0\), 有

\[f(0)+f(x)=f(0),\]

即, 对于任意的实数 \(x\), \(f(x)=0\) 为真.

如果 \(f(0)\ne0\). 在 \(f(x)\) 为符合要求的函数, 则 \(-f(x)\) 亦为所求. 故此, 我们只要考虑 \(f(0)\gt0\) 此种情况就行了.

令 \(x=y=0\), 得

\[f(f^2(0))+f(0)=f(0).\]

若记 \(f^2(0)=a\), 则 \(f(a)=0\).

如果 \(f(b)=0\), 则 \(b=1\).

若不然, \(b\ne1\). 令 \(x=\dfrac b{b-1}\), \(y=b\), 则

\begin{equation}f(f(\frac b{b-1})f(b))+f(\dfrac b{b-1}+b)=f(\frac {b^2}{b-1}),\end{equation}

即 \(f(0)=0\). 矛盾!  故而, \(f(a)=0\) 蕴涵 \(a=1\), 进而, 根据 \(f^2(0)=1\), 鉴于我们只观摩  \(f(0)\gt0\), 于是 \(f(0)=1\), \(f(1)=0\).

令 \(y=1\), 我们得出

\[f(0)+f(x+1)=f(x),\]

\begin{equation}f(x+1)=f(x)-1.\end{equation}

下面指出: \(f(x)\) 为单射.

事实上, 如果实数 \(a\) 和 \(b\) 使得 \(f(a)=f(b)\). 注意

\begin{equation}\big(a^2-4(b-1)\big)+\big(b^2-4(a-1)\big)=(a-2)^2+(b-2)^2\geqslant0\end{equation}

说明 \(a^2-4(b-1)\geqslant0\), \( b^2-4(a-1)\geqslant0\) 至少一个, 不妨前者, 为真.

然后, \(a^2-4(b-1)\geqslant0\) 蕴涵二次方程 \(x^2-ax+b-1=0\) 有两个实根 \(r\), \(s\). 故此, \(r+s=a\), \(rs=b-1\).

令 \(x=r\), \(y=s\),

\[f(f(r)f(s))+f(r+s) = f(rs).\]

根据 \((3)\),

\[f(f(r)f(s)+1)+f(r+s) = f(rs+1).\]

由 \(r+s=a\), \(rs+1=b\),

\begin{equation}f(f(r)f(s)+1) = 0\end{equation}

这表示 \(f(r)f(s)+1=1\), 即 \(f(r)f(s)=0\), 故 \(f(r)=0\), 即 \(r=1\),  或 \(f(s)=0\), 即 \(s=1\). 无论是 \(r=1\), 此时 \(a=b=s+1\), 还是 \(s=1\), 此时 \(a=b=r+1\), 都能得出 \(a=b\). 从而 \(f(x)\) 为单射.

令 \(y=-x\), 得

\[f(f(x)f(-x))+f(0) = f(-x^2).\]

这就是

\[f(f(x)f(-x)) = f(-x^2)-1=f(1-x^2).\]

\(f(x)\) 为单射导出

\begin{equation}f(x)f(-x) =1-x^2.\end{equation}

令 \(y=1-x\), 得

\[f(f(x)f(1-x))+f(1) = f(x-x^2).\]

这就是

\[f(f(x)f(1-x))= f(x-x^2).\]

\(f(x)\) 为单射导出 \(f(x)f(1-x)=x-x^2\), 即

\begin{equation}f(x)(f(-x)-1) =x-x^2.\end{equation}

于是

\[f(x)=f(x)f(-x)-(x-x^2)=(1-x^2)-(x-x^2)=1-x.\]

经检验, \(f(x)=1-x\), \(f(x)=x-1\), \(f(x)=0\) 满足条件, 从而就是我们要找的全部函数.

Problem 3 (Austria)

这个题刷新了记录, 成了 IMO 得分最低的题. 考场上居然只有 2 个人得到 7 分, 一共也只有 7 人的分不是 0, 尤其中国, USA 这样的竞赛强国在这个题都得了 0. 中国上一次出现这样的尴尬还是 21 年前, 即 1996 年的 P5 的几何不等式.

本题有专文处理: IMO 2017 solutions II

Problem 4 (Charles Leytem, Luxembourg)

写第 3 题花费不少时日, 第二天的题迟迟未动笔. 这个题只是寥寥数语.

IMO 2017

IMO 2017 p4

记 \(RA\) 与 \(\Gamma\) 的另一个交点为 \(B\). 连结 \(KR\), \(KS\), \(BS\), \(BT\).

\(K\), \(R\), \(J\), \(S\) 四点共圆, 以及\(S\), \(J\), \(A\), \(B\) 亦是四点共圆, 定出 \(\angle KRS=\angle KJS=\angle RBS\).

\(RA\) 为 \(\Omega\) 的切线蕴涵 \(\angle RKS=\angle BRS\). 于是, \(\triangle RKS\sim\triangle BRS\), 故此

\(\angle RSK=\angle BSR\), 然后  \(\angle RSK=\angle BSR\);

以及

\(\frac{KS}{RS}=\frac{RS}{BS}\),  结合 \(RS=TS\), 然后

\[\frac{KS}{TS}=\frac{TS}{BS}.\]

至此, 我们可以断言 \(\triangle KTS\sim\triangle TBS\), 进而 \(\angle KTS=\angle TBS\), 这也就导出了直线 \(KT\) 与圆 \(\Gamma\) 相切.

Problem 5 (Russia)

用归纳法几句话就能透彻.

把全部的队员按身高分成 \(N\) 组: 身高最低的 \(N+1\) 个队员一组, 身高次低的 \(N+1\) 个队员一组, …, 身高最高的 \(N+1\) 个队员一组. 我们来从每组选出  \(2\) 人, 使得在最后的  \(2N\) 人, 属于同组的  \(2\) 人是紧挨的.

\(N=2\) 时, 这排球员左边 \(3\) 人必有 \(2\) 人属同一组, 选出这 \(2\) 人; 右边 \(3\) 人必有 \(2\) 人亦属同一组, 选出这 \(2\) 人. 如此, 教练移走了 \(2\) 人, 剩下 \(4\) 人的左边  \(2\) 人与右边 \(2\) 人分属不同的组.

假定对于 \(N(N-1)\) 个球员结论为真, \(N\geqslant3\). 下面来考察 \(N(N+1)\) 个球员.

这一排最左边的 \(N+1\) 人必有 \(2\) 人是同一组. 在这\(N+1\) 人选出同组的 \(A\) 和 \(B\),  且 \(A\) 在 \(B\) 左边, 并且 \(B\) 左边的队员都属于不同的组. 移走 \(B\) 左边除 \(A\) 以外的其他队员, 再把 \(B\) 右边那些与 \(B\) 同组队员全部移走. 这样, \(B\) 右边的队员全部与 \(B\) 不同组, 并且 \(B\) 不属于的每个组至少留下 \(N\) 人在 \(B\) 右边. 如果有哪个组有 \(N+1\) 人在 \(B\) 右边, 就随便在这组移走 \(1\) 人. 现在, \(B\) 右边 \(N(N-1)\) 个球员分属于 \(N-1\) 组, 每组 \(N\) 人. 由归纳假设, \(B\) 右边的 \(N(N-1)\) 个球员可以每组选出 \(2\) 人是紧挨的. 这选出的 \(2(N-1)\) 人, 以及 \(A\) 和 \(B\), 这 \(2N\) 个队员属于同组的  \(2\) 人是紧挨的.

Problem 6 (John Berman, USA)

本题用来做 2 或者 5 是比较合适的.

对 \(|S|\) 进行归纳.

当 \(S\) 只有一个元素 \((p_0, q_0)\) 时, 既然 \((p_0, q_0)=1\) 时, 著名的 Bezout 恒等式指出存在整数 \(a\), \(b\), 使得

\[ap_0+bq_0=1.\]

令 \(P(x, y)=ax+by\), 此多项式对于\(S\) 的惟一的元素 \((p_0, q_0)\), 有 \(P(p_0, q_0)=1\).

假定当 \(|S|=m\) 时, 命题为真, \(m\geqslant1\). 下面认定 \(S = \left \{ (p_1, q_1), (p_2, q_2), \dotsc, (p_{m+1}, q_{m+1}) \right \}\), \(|S|=m+1\).

依归纳假设, 有齐次整系数多项式 \(G(x, y)\) 符合 \(G(p_k, q_k) = 1\), \(k=1\), \(2\), \(\dotsc\), \(m\). 令 \(\deg(G)=g\).

然后,  \((p_{m+1}, q_{m+1}) =1\), 故此, 存在整数 \(u\), \(v\), 使得

\begin{equation}up_{m+1}+vq_{m+1}=1.\end{equation}

考察

\begin{equation}F(x, y) = \big (G(x, y) \big )^h – w\Big(\prod_{k=1}^m\left ( q_kx-p_ky \right ) \Big) \Big(ux+vy\Big)^{gh-m},\end{equation}

这里的 \(h\) 是大于 \(\dfrac mg\) 的待定的正整数, \(w\) 是待定的整数. 于是, \(F(x, y)\) 是整系数齐次多项式, 并且 \(F(p_k, q_k) = 1\), \(k=1\), \(2\), \(\dotsc\), \(m\).

令 \(A= G(p_{m+1}, q_{m+1})\), \(B=\prod\limits_{k=1}^{m}\left(q_kp_{m+1}-p_kq_{m+1} \right )\). 下面来说明  \((A, B)=1\).

若不然, 存在质数 \(p\), 满足 \(p|G(p_{m+1}, q_{m+1})\), \(p|\left(q_kp_{m+1}-p_kq_{m+1} \right )\), 这里 \(k\in\{1, 2, \dotsc, m\}\).

由 \(\left(q_k, p_k \right )=1\), 因此 \(q_k\), \(p_k \) 必有一个不被 \(p\) 整除, 不妨 \(q_k\) 不被 \(p\) 整除. 既然 \(G(x, y)\) 是齐次多项式, 并且 \(p_kq_{m+1}\equiv q_kp_{m+1}\pmod p\), 于是

\begin{equation}q_{m+1}^g = q_{m+1}^g G(p_k, q_k) = G(p_kq_{m+1}, q_kq_{m+1}) \equiv G(q_kp_{m+1}, q_kq_{m+1}) = q_k^g G(p_{m+1}, q_{m+1}) \equiv 0 \pmod p\end{equation}

导出 \(p|q_{m+1}\). 结合 \(p|\left(q_kp_{m+1}-p_kq_{m+1} \right )\), 给出 \(p|p_{m+1}\). 矛盾!

既然 \((A, B)=1\), Euler 定理指出, 存在足够大的正整数 \(h\) 以及整数 \(w\), 使得

\begin{equation}A^h – wB = 1.\end{equation}

如此一来,

\begin{equation}\begin{split}F(p_{m+1}, q_{m+1}) &= \big (G(p_{m+1}, q_{m+1}) \big )^h – w\Big(\prod_{i=1}^m\left ( q_ip_{m+1}-p_i q_{m+1} \right ) \Big)\Big(up_{m+1}+v q_{m+1}\Big)^{gh-m}\\&=A^h – wB=1.\end{split}\end{equation}

这便完成了征途.

Annotations

  1. 第 2 题的函数方程, 没有什么新奇的. 得分那么低, 倒是有点出乎意料.
  2. 第二天的题其实没啥特别, 难度也不大. 题 6 可以推广为更普遍的形式.
  3. 第三题确实有独到之处, 是这个试卷仅有的好题.  可以对 \(m\) 个猎人考虑同样的问题.
  4. 这六个题何以成为史上得分最低的试卷呢! 除了第三题, 别的题为啥得分也不高?
  5. 大陆居然在函数方程载了跟头: 函数方程应该是必须掌握的基本功. 最难的题最可能来自组合, 这没有疑问; 数论的难题虽多, 但能成为竞赛卷子的妙题却不容易; 考场上的几何再难, 也没太大意思.
 Posted by at 2:24 pm  Tagged with: